If a line is perpendicular to y =-1/3x-5, what is the slope of that
A.-1/3
B.1/3
C.3
D.-3

Answers

Answer 1

Answer:

c

Step-by-step explanation:

the slope of a line perpendicular will always be the opposite

the slope of a line parrallel will always be the same


Related Questions

What is the slope-intercept equation for the line below?

A) y = 5/4x - 4
B) y = 4/5x - 4
C) y = -4/5x - 4
D) y = -5/4x - 4​

Answers

Answer:

A) y = 5/4x - 4

Step-by-step explanation:

Slope-intercept formula

y = mx + b, where m = slope, b = y-intercept

Slope formula

m = (y2 - y1)/(x2 - x1)

Given points (on the graph)

(0, -4) and (4, 1)

Slope is:

m = (1 - (-4))/(4 - 0) = 5/4

y-intercept is

-4 as per the first point

So the line is:

y = 5/4x - 4

Correct answer option is: A) y = 5/4x - 4

Answer:

it is a very very easy sum!

Step-by-step explanation:

goooooood afternoon

Systems by elimination:
please help me..ASAP

Answers

Answer:

#1 is no solution and #2 is infinitely many solutions

the first one would be c) (2,-9)
the second one i’m not sure

(WILL GIVE BRAINLIEST TO RIGHT ANSWER) a container holding water is 3/4 full. after pouring out 8 ounces it is half full. How many ounces can it hold? x = # ounces container can hold

Answers

it can hold 32 ounces

Answer:

If x is the number of ounces the container can hold then x = 32

Step-by-step explanation:

The container can hold 32 ounces because if it was 3/4 full and then you poured out 8 ounces then that means there was 2/4 remaining.

Hope this helps!!

50°
(x + 16)




Please help

Answers

Answer:

50o • (x + 16)

Step-by-step explanation:

it stays the same.

Write the equation y = (4/5 x) - 7/10 in standard form. (Standard form is Ax+By = C, where A is positive, and A, B, and C are integers with greatest common divisor 1.)

Answers

Answer:

8x-10y=7

Step-by-step explanation:

A ruby ring which normally sells for ​$310 is on sale for ​$217. Find the percent markdown for each ring.

Answers

Answer:

My awnser isss

Step-by-step explanation:

32% my homie

Answer:30%

Step-by-step explanation:

Discount = Original Price (X) discount (X) %/100

Discount = 310 × (30)/100

Discount = 310 x 0.3

You save = $93.00

What is the measure of angle 1?
Write the answer without the degree.

Answers

Answer: 70

90+20=110

180-110=70

I hope this is good enough:

Answer:

70

Step-by-step explanation:

Everything needs to add up to 180. So you have 90 and 20 you will add them and you’ll get 110. Since you want to find the missing measurement you will subtract 180-110 and you’ll get 70. The missing angle is 70. To check: 90+20+70=180

Eddie deposits $1090 in an account that earns 6% interest compounded annually for 3 years. What is the total interest earned after 3 years? Round to the nearest cent.
A: 196.20
B: 208.20
C: 1286.20
D: 1298.20

Answers

Answer:

Step-by-step explanation:

Eddie deposits $1090 in an account that earns 6% interest compounded annually for 3 years. What is the total interest earned after 3 years? Round to the nearest cent.

A: 196.20

B: 208.20

C: 1286.20

D: 1298.20

How do I make someone brainlest?

Answers

Answer:

When 2 people  answers a question you should see a crown and you click  on it and  that is how you marke someone brainlest

Step-by-step explanation:

4. John is packing books into boxes. Each box can hold either 15 small books or 8
large books. He needs to pack at least 35 boxes and at least 350 books, Write a
system of linear inequalities to represent the situation

Answers

Answer:

Your system of inequalities is:

x + y ≥ 35

15x + 8y ≥ 350

Step-by-step explanation:

Let x = number of boxes of small books and let y = number of boxes of large books.

 Since John needs to pack at least 35 boxes:

 (number of boxes of small books) + (number of boxes of large books) (is at least) 35

x + y ≥ 35

Since John needs to pack at least 350 books:

 (number of small books packed) + (number of large books packed) (is at least) 350

(number of small books per box)(number of boxes of small books) + (number of large books per box)(number of boxes of small books) ≥ 350

15x + 8y ≥ 350

A movie theater charges $9.00 for adults and $7.00 for senior citizens. One a day when 325 people
paid an admission, the total receipts were $2495. How many who paid were adults? How many were
seniors? (please show your work)

Answers

Answer:

325 x 7.00 = 2275.

2495 - 2275 = 220.

Solution set of equation x^2-5x=0

Answers

♥️♥️♥️♥️♥️♥️♥️♥️♥️♥️♥️♥️♥️♥️

[tex] {x}^{2} - 5x = 0[/tex]

Factorise x

[tex]x(x - 5) = 0[/tex]

♥️♥️♥️♥️♥️♥️♥️♥️♥️♥️♥️♥️♥️♥️

Hint :

[tex]a \times b = 0[/tex]

[tex]a = 0 \: \: \: \: or \: \: \: \: b = 0[/tex]

♥️♥️♥️♥️♥️♥️♥️♥️♥️♥️♥️♥️♥️♥️

Thus ;

[tex]x = 0 \: \: \: \: or \: \: \: \: x - 5 = 0[/tex]

So :

[tex] x = 0 \: \: \: \: or \: \: \: \: x = 5[/tex]

♥️♥️♥️♥️♥️♥️♥️♥️♥️♥️♥️♥️♥️♥️

CHECK :

[tex]x = 0[/tex]

[tex] {0}^{2} - 5(0) = 0[/tex]

[tex]0 - 0 = 0[/tex]

[tex]0 = 0[/tex]

The solution is correct.

♥️♥️♥️♥️♥️♥️♥️♥️♥️♥️♥️♥️♥️♥️

[tex]x = 5[/tex]

[tex] {5}^{2} - 5(5) = 0[/tex]

[tex]25 - 25 = 0[/tex]

[tex]0 = 0[/tex]

The solution is correct.

♥️♥️♥️♥️♥️♥️♥️♥️♥️♥️♥️♥️♥️♥️

I think A or b will equal 0

Is (1, -1) a solution to y = 3x - 4?
Explain.

Answers

(1,-1) is a solution to y=3x-4 because when we plug in 1 for x in the equation we get y=-1. Since x is 1 and y is -1 we can see that the coordinate points (1.-1) are a solution to y=3x-4.

Please Mark me as Brainliest!

Answer:

yes

Step-by-step explanation:

just plug in the point into the equation then simplify

A bus travels at a speed of 40 miles per hour and reaches Hogville in 40 minutes. If a cab takes 35 minutes to reach the same spot, determine the speed at which it traveled.

Answers

Answer:

45.7miles per hour

Step-by-step explanation:

40mins = 2/3 hour

the bus travelled (2/3 x 40) miles = 80/3 miles

the cab took 35 mins (which is 7/12 hour) so it's speed is (80/3)/(7/12) miles/hour = 45.714 miles/hour (5 s.f.)

What is the volume of the prism below?
Aw
2
23
units
$3
3. units
5 units 3
A units 3

Answers

Answer:

V = 3.5 m3

Step-by-step explanation:

length l = 1 m

width w = 2 m

height h = 1.75 m

volume V = 3.5 m3

Solve the right triangle shown in the figure. (Round your answers to two decimal places.)

Answers

Answer:

Step-by-step explanation:

(3). cos A = [tex]\frac{b}{c}[/tex]

cos A = [tex]\frac{1.77}{17.7}[/tex] = 0.1 ⇒ A ≈ 84.26°

B ≈ 5.74°

C = 90°

a ≈ 17.61

(4). cos 16° = [tex]\frac{79}{x}[/tex] ⇒ x ≈ 82.18 ≈ 82.2 ≈ 82

Z is the circumcenter. What is VZ?

Answers

Answer:

VZ is 24

Step-by-step explanation:

when we talk of the circumcenter, we mean a point in the triangle that has a uniform distance from each of the vertices of the triangle

What we mean by the circumcenter is that the length between this point and any of the vertices is the same distance

ZU is a distance between the circumcenter and the vertice

What we want to calculate top is same.

So therefore, the two are equal and we have the length of what we want to

calculate as 24

need rn! worth 13 ~ what is the equation of the parabola? 謝謝

Answers

Answer:

Its B.

Step-by-step explanation:

Emath.com- Parabola calculator

PLZZZZZZ HELP ME!!!!!!!

15 x 5x

Answers

75 are u that dumb boy

Answer: x=0

Step-by-step explanation:

x in (-oo:+oo)

((15*x)/5)*x = 0

3*x^2 = 0 // : 3

x^2 = 0

x = 0

x = 0

2-5/6x=-5

what is x
pls show me how to solve it

Answers

x=8.4 work is in the picture box.

What is the answer to answer to 7m-3m-6<6?

Answers

Answer:

m<3

Step-by-step explanation:

This the answer to the equation.

The cost of 2 cups of strawberries is $6. Write an equation for the relationship between the cost and cups os strawberries

Answers

Answer:One equation is x=6/2

Hope this helps

Answer:

the strawberrys are worth $3 per cup.

Step-by-step explanation:

s= $6

-6 = 2/3r solve for r

Answers

r=-9


explanation: because

Answer:

r=9

Step-by-step explanation:

The ratio of Braydon's number of laps he ran to the time he ran is 6:2. The ratio of Monique's number of laps she ran to the time she ran is 104. Explain why these ratios are not in a proportional relationship.​

Answers

Answer:

They are not in a proportional relationship because Monique's number and time isnt a proportion. Instead it is a whole number. Like Braydon's his is 6:2 that is the correct way to write it.

Step-by-step explanation:

The ratio is not proportion.

What is ratio and proportion?

A ratio is an ordered pair of numbers a and b, written a / b where b does not equal 0. A proportion is an equation in which two ratios are set equal to each other.

Given:

The ratio of Braydon's number of laps he ran to the time he ran is 6:2.

The ratio of Monique's number of laps she ran to the time she ran is 10:4.

The simplest for Braydon is = 6/2 = 3/1

and, the simplest form for Monique = 10/4 = 5/2

As, 3/1 ≠ 5/2

So, the ratio is not proportion.

Learn more about proportion here:

https://brainly.com/question/26974513

#SPJ2

I will give brainliest and give 17 points please help!!

Answers

Answer:

1. New England Patriots: (3,-2)

2. Miami Dolphins: (-2,1)

3. Denver Brancos: (4,3)

4. Green Bay Packers: (-1,3)

5. San Franciso 49ers: (-4,-4)

6. Pittsburg Steelers: (2,2)

7. Baltimore Ravens: (3,1)

8. Dallas Cowboys: (4,-3)

9. Oakland Raiders: (-4,2)

10. Washington Redskins: (-3,-1)

Stephen has a dog that weigths 5 times as much as lan's dog.The total weigths of both dogs is 72 pounds.How much does Stephens dog weigths?

Answers

Answer: Stephens dog weighs 60 pounds

Step-by-step explanation:

Can someone put this into vertex form?
y=x2+24x-15

Answers

Answer:

The vertex form of y = x² + 24x - 15 is y = (x + 12)² - 159

Step-by-step explanation:

The vertex form of the quadratic equation y = ax² + bx + c is

y = a(x - h)² + k, where

a is the coefficient of x²(h, k) are the coordinates of the vertex pointh = [tex]\frac{-b}{2a}[/tex] , where b is the coefficient of xk = y at x = h

Let us use the facts above to solve the question

y = x² + 24x - 15

∵ a is the coefficient of x² and b is the coefficient of x

a = 1 and b = 24

∵ h is the x-coordinate of the vertex point

∵ h = [tex]\frac{-b}{2a}[/tex]

→ Substitute the values of a and b to find h

∴ h = [tex]\frac{-24}{2(1)}=\frac{-24}{2}[/tex]

h = -12

→ To find k substitute y by k and x by h

∵ y = k and x = h

∴ k = (-12)² + 24(-12) - 15 = 144 - 288 - 15

k = -159

∴ The coordinates of the vertex point are (-12, -159)

→ Substitute the values of a, h, and k in the vertex form above

∵ y = 1(x - -12)² + (-159)

→ Remember (-)(-) = (+) and (+)(-) = (-)

y = (x + 12)² - 159

The vertex form of y = x² + 24x - 15 is y = (x + 12)² - 159

Answer:

sure.

Step-by-step explanation:

Q is the midpoint for the points S(11, -6) and T(-1, 10). Find the length of segment QT.
QT=

Answers

Answer:

10

Step-by-step explanation:

QT=ST/2=√(12^2+16^2)/2=10

.
HELPPP
The equation of the line in slope Intercept form is

Answers

Answer:

y = 60x + 40

Step-by-step explanation:

It is in the form of

y = mx + b

m is the slope

b is the y intercept.

The y intercept is 40, so it will be y = mx + 40.

The slope is 60, so y = 60x + 40!

Francisco played a video game and earned 60 points for every coin he collected he collected 7 coins

how many points did he earn for the coins that he collected? there are no Onnline answers

Answers

Answer:

420 points

Step-by-step explanation:

multiply 7 by 60

4,200 because you multiply 60 by 7 to get your anwser
Other Questions
Does anyone know how to do this?!? Provide the appropriate forms of the adjectives The cost of a gym membership is $25 per month. There is an initial fee of $40 What are the effects of the more commonly used psychoactive drugs? 1. W ktrych zdaniach wyrnione zakoczenia zapisano niezgodnie z wymow? Podaj wyrazypokrewne uzasadniajce tak pisowni. Prowadz prywatne ledztwo, aby dowiedzie si, kim jestem.Ludzie przypominaj czasem dzikie ptactwo. Nic, co ludzkie, nie jest mi obce.. Games Galore Super Store buys the latest video game at a wholesale price of $58.00. The markup rate at Games Galore Super Store is 40%. You use your allowance to purchase the game at the store. How much will you pay, not including tax? The Populist Party in Georgia was more like to attract?wealthy northernersabolitionistssmall farmersfederal bureaucrats Help me? on a timer tysvm don't rush just need the correct answer. Do someone know the answer 1 point15 The Davenports had a repairman come out totheir house. They were charged $50 per hour forthe 2-hour visit, plus a $99 consultation fee.What was the total charge?$help ASAP!Your answer Why is part of the Moon dark? A)Light from the sun only illuminates half of the Moon. B)Earth's shadow is on half of the Moon. C)The Moon only makes light on one side. D)Clouds are blocking half of the Moon's light. E)The Moon is made up of light and dark rocks. What will most likely happen if you place a piece of metal in a base?O A. The metal will turn into a salt.O B. The metal will neutralize the base.C. The metal will give off a gas.O D. The metal will not react with the base. Before he buys a new smartphone, Tim can best use data toevaluate which phone he should get.convince himself he wants a phone.discredit others opinions on his phone choice. recognize problems that all phones have. Renting video games from Store S costs $2.50 per game plus a monthly fee of $5.00. Renting video games from Store T costs $5.00 per game with no monthly fee. The monthly cost to rent video games depends on the number of video games, v, rented. Which inequality represents the situation when the monthly cost at Store S is less than the monthly cost at Store T?A. 2.5v + 5 < 5vB. 2.5v + 5 = 5vC. 7.5v < 5vD. 7.5v = 5v 7th grade help me plzzzzzzzzzz What does the Primary process do? What are the major steps to getting nominated by your political party? uh i got put into algebra 1 pls help If the starting population of 5 rabbits grows at 200% each year, how many will there be 50 years? What are some of the medical advances made recently?MRISCamera's small enough to swallowSterilization of instrumentsEye glassesPenicillinCAT scans The areas that bordered the Eastern Mediterranean were collectively called __________. A. Israel B. Palestine C. the Levant D. the Middle East Urgent!!!!! Please answer!!!!!!